Talk:2021 Fall AMC 10A Problems/Problem 19

Revision as of 13:08, 25 November 2021 by Mathfun1000 (talk | contribs) (A bit of help)
(diff) ← Older revision | Latest revision (diff) | Newer revision → (diff)

An asymptote diagram for my solution would be appreciated. You can take credit for the diagram, but please write it in the form: ~username (Diagram)

Note: I can not do asymptote diagrams.